LSAT and Law School Admissions Forum

Get expert LSAT preparation and law school admissions advice from PowerScore Test Preparation.

User avatar
 Dave Killoran
PowerScore Staff
  • PowerScore Staff
  • Posts: 5852
  • Joined: Mar 25, 2011
|
#44132
Complete Question Explanation
(The complete setup for this game can be found here: lsat/viewtopic.php?t=8628)

The correct answer choice is (D)

Since a pair of parakeets must be exhibited, and answer choices (A) and (C) do not contain a pair of parakeets, both answer choices can be eliminated. Answer choice (B) can be eliminated since J and S cannot be exhibited together. Answer choice (E) can be eliminated since W and S cannot be exhibited together. It follows that answer choice (D) is correct.
 ccampise
  • Posts: 27
  • Joined: Jul 14, 2014
|
#15445
What about the question in 14 conveys that a pair of parakeets must be selected? Two pairs could mean one pair of parakeets and one pair of goldfinches or lovebirds couldn't it?
 Emily Haney-Caron
PowerScore Staff
  • PowerScore Staff
  • Posts: 577
  • Joined: Jan 12, 2012
|
#15446
Hi ccampise,

The way I'd approach this one is by taking one rule at a time to see which answer choices it eliminates. The last rule is the easiest to start with; J or W can't be with S, which knocks out answers B and E. Additionally, when you consider the second rule, given that we have 3 male parakeets, we know at least one pair of parakeets has to be exhibited. That knocks out answers A and C, neither of which have a parakeet pair listed. The answer, therefore, is D.

I hope that helps!

Get the most out of your LSAT Prep Plus subscription.

Analyze and track your performance with our Testing and Analytics Package.